LSAT and Law School Admissions Forum

Get expert LSAT preparation and law school admissions advice from PowerScore Test Preparation.

 PB410
  • Posts: 39
  • Joined: Apr 01, 2017
|
#37707
Hi,

is answer choice C wrong because the stimuli states "But if some peoples are peaceable now, then aggression itself cannot be coded in our genes, only the potential for it" is used as support for the conclusion and not the conclusion itself? Acting as support to the conclusions, it would be inaccurate to claim that the author is "arguing that there are no genetically based traits"?
 Eric Ockert
PowerScore Staff
  • PowerScore Staff
  • Posts: 164
  • Joined: Sep 28, 2011
|
#37723
Answer (C) is incorrect because it is too strongly worded. The author never actually argued that there are "no genetically based" traits. The author cites the presence of peaceable people as evidence that AGGRESSION isn't coded into our genes. But that leaves the door open for any number of possible traits that still could be genetically based.

Remember that Method of Reasoning questions always have this Prove Family standard built into them. Whatever answer you select must be provably true based on the stimulus. So, much like Must Be True questions, you will often encounter answers like answer (C) that take things a little too far with exaggerated language. It keeps the answer close enough to the stimulus to be tempting, but it's ultimately a statement that goes beyond what you can prove about the author's argument.
 alberto
  • Posts: 30
  • Joined: Aug 29, 2018
|
#68172
Its Alberto.

Preptest 23 Oct 1997, section 3 question #8 logical reasoning where the answer is B.
The force of the concept of innateness used in the argument...... so on.

Why force?
I did not see the synonym in the stimulus to represent force,and only words used prevalence and disinclination. The argument seems tame passive. Not forceful, the author's point.

I guess that is why it's the weaken answer?
I made mistake and chosen A. The accuracy of the historical data cited in the argument for aggressiveness is called until question. Both answers A and B mentioned the last statement: calls into question, one helps to strengthen and the other weakens. I think I get it , however, if time permits please explain? Thanks.

Alberto
 James Finch
PowerScore Staff
  • PowerScore Staff
  • Posts: 943
  • Joined: Sep 06, 2017
|
#68310
Hi Alberto,

First thing, this isn't a strengthen or weaken question, but a method one, in which we are asked to describe how the stimulus attacks the the claim mentioned in the first sentence. With that out of the way, (A) doesn't work because it isn't in the stimulus; the stimulus seems to agree with the historical data, and certainly doesn't attack it. What it does do is attack the definition of the word "innate," which is what we should be Prephrasing for this question. (B) is the only answer choice that speaks to this, making it correct.

Hope this helps!

Get the most out of your LSAT Prep Plus subscription.

Analyze and track your performance with our Testing and Analytics Package.